59
8/11/2019 2003 step 1 http://slidepdf.com/reader/full/2003-step-1 1/59  2003 USMLE STEP 1 REMEMBERED QUESTIONS © USMILE.US 

2003 step 1

Embed Size (px)

Citation preview

Page 1: 2003 step 1

8/11/2019 2003 step 1

http://slidepdf.com/reader/full/2003-step-1 1/59

 

2003 USMLE STEP 1 REMEMBERED QUESTIONS

© USMILE.US 

Page 2: 2003 step 1

8/11/2019 2003 step 1

http://slidepdf.com/reader/full/2003-step-1 2/59

© usmile.us 

what is the second messenger of secretin?cAMP opposite to CCK, that is why they potentiate each other

what is the second messenger of CCK?IP3&increased intracellular Ca++

which condition associated with no change in Hct level?a)diarrheab)SIADHc)adrenal insufficiencyd)isotonic NaCl infusionb.SIADH- hypoosmotic expansion. so ECF increases ,and RBS conc should decreses, but RBSswell, so no changes in the conc.

which effect is not seen with thiazide diuretics?a)increased K+ excretionb)decreased Ca++ excretionc)decreased ability to dilute urined)decreased ability to concentrate urined.No effect on ability to concentrate

which statement about gastrin is incorrect?a)gastrin stimulates growth of mucosa of the small intestine and colonb)the most potent releasers of gastrin are phenylalanine and tryptophanc)atropine blocks vagus-mediated gastrin released)stomach distention stimulates gastrin secretionc.Atropine does not block,since the mediator is GRP not Ach

high grade malignant Bcell lymphoma that most often develops in the setting of abnormalimmune states(e.g SLE,Sjogren syndrome.......)a)follicular lymphomab)small lymphocytic lymphomac)lymphoblastic lymphoma

d)large cell immunoblastic lymphomad)large cell immunoblastic lymphoma

which one is not a stimulus for release of CCK?a)small peptidesb)amino acidsc)fatty acidsd)triglyceridesd,triglycerids:bcoz they can’t cross the intestinal mucosal membrane,but monoglycerides cancross& stimulate CCK secretion

which one causes hypokalemia?a)insulin deficiency

b)hyposmolarityc)digoxind)triamterenee)beta-adrenergic antagonistsb:hyposmolaritycoz H2O flows into the cells,K+ diffuses in with H2O leads to hypokalemiaall other choices with hyperkalemia, remember that digoxin inhibits Na+/K+ pump causeshyperkalemia

 AND hypokalemia predisposes to digoxin toxicity!

Page 3: 2003 step 1

8/11/2019 2003 step 1

http://slidepdf.com/reader/full/2003-step-1 3/59

© usmile.us 

autosomal recessive childhood cyctic disease that's bilateral and associated withpolyuria,growth retardation,the presence of corticomedullary cysts and cortical atrophy of tubules:a)medullary sponge kidneyb)medullary cystic diseasec)childhood polycystic kidney diseased)cystic renal dysplasiathis is a case of medullary cystic disease

virulance factor of mycobacterium TB that inhibit phagosome-lysosome fusion?SulfolipidsM . tb survives in a phagosome in the cell & produces a protein called "exported repetitiveprotein" which prevents phagosome & lysosome from fusing

it determines the percent saturation of Hgb?a)PaO2b)PaCO2c)Hgb concenterationd)both a&cthe answer is PaO2

after 6mo of drug therapy,a patient develops HTN&creatinuria without proteinuria.A biopsyshows ischemic damage to glumeroli and some tubular damage:a)penicillinGb)gentamicinc)cyclosporined)naproxene)cyclophosphamidecyclosporine. it causes nephrotoxicity & hypertension.....pen g- causes acute hypersensitive interstitial nephritis..., gentamicin causes toxic ATN but thatwould be acute too & it would involve the tubule mainly...naproxen may cause chronic analgesic nephropathy.. but that would first manifest as papillarynecrosis..& cyclophosphamide causes cystitis....

this condition makes no change in filtration fraction:a)constriction of afferent arterioleb)constriction of efferent arteriolec)increased plasma proteind)ureteral stonea. in uretheral stone and increased plasma protein- RPF- is unchanged, but GFR- changed, soFF is also changed. in efferent artetiole, they change in opposite, so FF also changed., only inafferent artetiole GFR and RPF change in the same direction - so FF is unchanged.

after several days of treatment the patient develops significant proteinuria,renal biopsyshows myeloid bodies in proximal tubule cells,what's the drug responsible?a)penicillinGb)gentamicin

c)cyclosporined)naproxene)cyclophosphamidemyeloid bodies in proximal tubule characteristic for gentamicin

which one is not a marker for ECF volume measurement?a)sulfateb)evans bluec)inulind)mannitol

Page 4: 2003 step 1

8/11/2019 2003 step 1

http://slidepdf.com/reader/full/2003-step-1 4/59

© usmile.us 

the answer is B:evans blue is a marker to measure plasma volumesulfate,inulin,mannitol to measure ECF volume

which effect is not seen in diarrhea?a)decreased ECF volumeb)increased ECF osmolarityc)increased Hctd)no change in ICF volumeb. in diarrhea lost both water and electrolytes, so ecf volume decreases, but no change in ecf andicf osmolarity. the result of decreased volume- increased Hematocrit, but RBS neither shrink norswell.

diarrhea is an example of:a)isosmotic volume expansionb)isosmotic volume contractionc)hyperosmotic volume expansiond)hyperosmotic volume contractione)hyposmotic volume expansionf)hyposmotic volume contractionb)isosmotic volume contraction

this hormone inhibits the effect of gastrin on growth of gastric cells:a)secretinb)CCKc)GIPd)somatostatinsecretin

cisapride increases GI motility by activating these serotonin receptors:a)5HT1Db)5H2c)5H3d)5H4

5HT-4

this disease transmitted by the same tick that transmits lyme disease?a)Babesiosisb)Relapsing feverc)Rocky Mountain spotted feverd)Ehrlichiosise)Colorado tick bite feverbabesiosis transmitted by ixodes tick

retrograde ejaculation is a side effect of:a)clonidineb)propranolol

c)prazosind)minoxidile)guanethidinef)captoprilguanethidineit’s an adrenergic blocker that inhibits sympa activity responsible for ejaculation

this drug acts by blocking guanosine monophosphate formation:a)ribavirinb)acyclovir

Page 5: 2003 step 1

8/11/2019 2003 step 1

http://slidepdf.com/reader/full/2003-step-1 5/59

© usmile.us 

c)gancyclovird)amantadinee)ziduvidineribavirin

what’s the effect of hyperventilation on serum Ca level?hyperventilation leads to respiratory alkalosis,elevated PH increases negative ions on albuminthat bind more to ionized Ca++,so ionized Ca++ decreases that leads to tetany but the amount oftotal serum Ca unchanged

the indication of use of Ames test?to identify potential carcinogens by assessing their ability to cause mutations

gr+ protozoan associated with gay bowel syndrome and drug of choice for treatment?Microsporidium,albendazole

treatment of cough due to captopril?u can prescribe aspirin, nifedipine, chromolyn sodium to treat coughchange to losartan another option(an angiotensin II receptor antagonist. It is not an ACE inhibitorand does not cause caugh)

For which of the following patients niacin can be a good choice to contol hyperlipidemia?a)a 47yr old male with history of goutb)a 56yr old female with history of NIIDMc)a 50 yr old male with history of angina pectorisd)a 38yr old male with history of duodenal ulcere)none of aboveit's Cniacin side effects:hyperglycemia,hyperuricemia,increased PUD(peptic ulcer)

which type of hypolipidemia can be associated with retinitis pigmentosa?abetalipoproteinemia that associated with retinitis pigmentosa

U can’t prescribe ketoconazole and amphotericinB together?why? Absorption of which one of these drugs depends on gastric PH?ketoconazole absorption depends on gastric PH&inhibited by H2 blockers and antacidsu can't prescribe these 2drugs together as one drug inhibits membrane ergosterolsynthesis(ketoconazole):fungicidal,so no fungus available for the action of ampho B that act byincreasing the permeability of fungal membranes:static effect

2 drugs that increase warfarin effect by displacing it from albumin?sulphonamides....also furosemide, chloral hydrate

which one is a risk factor for osteoporosis?smoking?alcohol?caffeine?smoking a RF for osteoporosiscaffeine not approved that be a RF

and low to moderate use of alcohol a protective effect on osteoporosis development

what’s the mode of inheritance of retinitis pigmentosa? AR-40%, AD-20%, X-linked-5%no herad of mitochondrial inheritence, could u tell me ur ref?or mitochondrial

a 50yr old male always arrives 15minute late for his appointment,which personality disorderhe has?a)schizoid

Page 6: 2003 step 1

8/11/2019 2003 step 1

http://slidepdf.com/reader/full/2003-step-1 6/59

© usmile.us 

b)borderlinec)histrionicd)passive-aggressivee)obcessive-compulsiveI think it's D because it's a possibilitythat the patient wants to show his unhappinessvia coming late all the time instead ofconfronting. Again, it could be E as well onlyif there was an explanation in the questionthat the patient comes late to appointmentbecause he was preoccupied to lock his housedoor many times.

amiodarone belongs to which class of antiarthmytic drugs?AND what's 3 important druginteractions of this drug?both class IA, III(K+ CHANNEL BLOCKER)increased level of phenytoin,warfarin,digoxin, increase theophylline and quinine levelsIt causes pulmonary fibrosis, corneal deposits, hepatotoxicity, both hypo&hyperthyroidism,blueskin discoloration due to iodine deposition as a side effect

which one is not a side effect of heparin overdose?a)hyperkalemiab)hypoglycemiac)hyperlipidemiad)thrombocytopeniathe answer is bhyperkalemia,hyperlipidemia,thrombocytopenia,osteoporosis side effects of heparin

anti-TB drug that can cause tubulointerestitial nephritis?Rifampin

diuretic associated with tubulointerestitial nephritis as a side effect?as per goljan both thiazides n furosimide causes tubulointerstitial nephritis

active toxic metabolite of cyclophosphamide that’s responsible for hemorrhagic cyctitis? Acrolein

active toxic metabolite of cyclophosphamide that’s responsible for hemorrhagic cyctitis? Acrolein

drug of choice for treatment of pinworm infection?mebendazole or pyrantel pamoate

which one is not a side effect of interferon therapy?a)pancytopeniab)confusion

c)thyroiditisd)upper GI irritation and ulceratione)fever,chillsd)upper GI irritation and ulcerationinterferon therapy sideeffects:fever,chills,myalgia,fatigue,pancytopenia,somnolence,confusion,thyroiditis

which one is not a side effect of cyclosporine?(may be more than one answer)a)hyperkalemiab)tremor

Page 7: 2003 step 1

8/11/2019 2003 step 1

http://slidepdf.com/reader/full/2003-step-1 7/59

© usmile.us 

c)seizured)gingival hyperplasiae)hypertensionf)hirsutismg)pancreatitish)glucose intolerancei)hepatic dysfunctionall are side effects of cyclosporine except g:pancreatitis

the mechanism of action of sibutramine?appetite suppression,blocking central serotonin,norepinephrine reupatakeSibutramine is a new agent used to treat obesity. It interferes with the reuptake of serotonin andNE into the presynaptic nerve terminal, thereby increasing their levels in the brain.

a child can use toothbrush and has imaginary friendat what age?4 yrs

deQurvein thyroiditis associated HLA?HLA-B35

chronic use of this drug to treat dermatitis herpetiform may lead to metheoglobinemia?Dapsone

the most common cause of vision loss in African-American population?open-angle glaucomaThey are five times more likely to develop it than whites, due to inherited risk factors. African-

 Americans over 40 and people in other ethnic groups over 60 are at greatest risk for developingglaucomaTrachoma is found in poor rural areas of most African countries and a few other areas today.Q is about African Americans.Cataract is the number 1 world wide cause of blindness.

 African Americans suffer 5x more from Glaucoma than other counterparts.

 Age-related macular degeneration (ARMD) is the number 1 cause of blindness after 60. So hereit is age related question.

drug of choice for treatment of sleep apnea?Protryptilline

what’s the second messenger for muscarinic receptors?(M1?M2?M3?)M1,M3:IP3M2:decreased cAMP

the indication of use of oseltamivir?the most common side effect?neuroaminidase inhibitors,block release of progeny virus from cellstreatment of influenzaA,B,vomiting in 1-2days that occurs with first dose and disappears with

continuing of treatment,other side effects:conjunctivitis,epistaxis,abdominal pain

the mechanism of action of zanamivir?indication of use?neuroaminidase inhibitors,block release of progeny virus from cells,treatment ofinfluenzaA,B,vomiting in 1-2days that occurs with first dose and disappears with continuing oftreatment,other side effects:conjunctivitis,epistaxis,abdominal painexacerbate pulmonary symptoms in COPD

what's the second messenger for histamine receptors?H1:IP3,H2:cAMP

Page 8: 2003 step 1

8/11/2019 2003 step 1

http://slidepdf.com/reader/full/2003-step-1 8/59

© usmile.us 

what's the second messenger for dopamine receptors?D1, 4 and 5 - cAMP, D2 and 3 -cAMP inhibition

what's the second messenger for alpha(alpha-1,alpha-2) receptors?alpha 1 IP3alpha 2 cAMP inhibition

the origin of gastrinoma in Zollinger-Ellison syndrome?a)alpha cellsb)beta cellsc)delta cellsthe answer is delta cells

the effects of Epinephrine and Norepinephrine on TPR(Total Peripheral Resistance)?NE:INCREASED TPR: act through alpha receptorsEP:DECREASED TPR: act through beta receptors

enzyme defect in porphyria cutanea tarda?partial loss of activity of hepatic uroporphyrinogen decarboxylase

enzyme defect in acute intermittent porphyria?defeciency in porphobilinogen deaminase(another name: uroporphyrinogen I synthase)

which part of thalamus considered a part of limbic system?ant thalamusmamillary bodies..........> ant.thalamaus.........>cingula

which one is not a derivative of middle trunk brachial plexus?a)median nerveb)ulnar nervec)radial nerved)axillary nerve

upper trunk:musculocutaneus nervemiddle trunk:median,radial,axillary nerveslower trunk:ulnar nerve

the origin of C cells of thyroid?a)first pouchb)second pouchc)third pouchd)4th pouche)5th pouch5th pouch(Dr Carl’s book)high yield & kaplan say 4th pouch... which forms the ultimobranchial body... (the actual origin ismigration of neural crest cells).

a patient with aluminium toxicity presents with dyspnea.why?(the cause of dyspnea?) Al toxicity leads to impaired absorption of P in GI tract,decreased available P leads to decreasedlevel of 2,3DPG,abnormal tissue oxygenation causes malaise,dyspnea,muscle weakness

the most common clinical presentation of chronic arsenic poisoning?neurologic symptoms predominate:burning sensation(glove-stock distribution),bilateral arm,legweakness(pigmentation another symptom but not the most common)

risk factors for developing tardive dyskinesia?

Page 9: 2003 step 1

8/11/2019 2003 step 1

http://slidepdf.com/reader/full/2003-step-1 9/59

© usmile.us 

Chronic use of Neuroleptis medications,CO poisoning,female sex etc also DM,old age,smoking

mechanism of action of nevirapine and common side effects?anti retro viral.... nonnucleoside reverse transcriptase inhibitor...which doenot require metabolicactivation..side effect is rash ... steven johnsons syndrome

mechanism of action of nelfinavir and common side effects?anti retro viral.. protease inhibitor...side effect diarrhea... central adiposity...inslin resistanceincreased levels of TG,cholestrol,lipodystrophy

to treat tricyclic antidepressant overdose which treatment is not effective and WHY?a)gastric lavageb)use of activated charcolc)physostigmined)dialysisd:dialysiscoz TCAs have a wide Vd(volume of distribution),so dialysis not effective

what's the difference betwen clinical presentation of NMS and serotonin syndrome?both of them present with hyperthermia, myoclonus,rigidity, CVinstabilityin additionNMS=altered consciousness is seenSErotonin syndrome=diaphoresis n seizures are seen

which of the following statistical tests is most appropriately used to evaluate the difference inthe percentage of women who lose weight on a protein-sparing diet versus the percentage wholose weight on a high-protein diet?a)paired t-testb)ANOVAc)chi-square testd)correlation

e)independent t-testchi square used to determine between frequencies in a sample

which of the following statistical tests is most appropriately used to evaluate the differencebetween initial body weight and final body weight for each woman on a protein-sparing diet?a)paired t-testb)ANOVAc)chi-square testd)correlatione)independent t-test

 A,t-test used to examine differences between means of 2 samples,this is paired t-test coz thesame women are examined on 2 diff occasionse)independent t-test:2groups of subjects are sampled on one occasion

an IQ test has high interrater reliability,what does it mean?it means that the results are very similar when the test is administered by a different examiner

these cells contain a peripheral hyalomere and central granulomere?a)lymphocytesb)monocytesc)basophilsd)neutrophilse)eosinophils

Page 10: 2003 step 1

8/11/2019 2003 step 1

http://slidepdf.com/reader/full/2003-step-1 10/59

© usmile.us 

f)plateletsplatelets!

in a kidney donor,what changes are seen in:1)creatinine excretion?2)GFR?3)plasma Cr concentration?Cr excretion=GFR x plasma Cr concentrationGFR decreased,plasma Cr increased,Cr excretion unchanged

which enzyme responsible for secretion of pancreatic enzymes?CCK or secretin?CCK responsible for pancreatic enzyme secretionsecretin for HCO3- secretion

Sabin-Feldman dye test used to identify this organism?Gold standard serologic test for toxoplasmosis

where’s the location of folic acid and iron absorption?yes,duodenum, according to kaplan most of folic acid and iron absorption occurs in duodenum

defeciency of this factor associated with thrombosis rather than hemorrhage?a)factor Vb)factor VIIc)factor XIId)factor XIIIfactor XII

intramembranous deposits are seen in which type of glomerulonephritis?MPGN typeIIMPGN typeI:with subendothelial deposits

coagulation factors inhibited by proteinC?factor V,VIII

which phase of cell cycle affected by tamoxifen?G1

In chemotherapy,what’ the only antibiotic that’s cell-cycle specific?and which phase ofcycle affected by this drug?0bleomycin--G2 phase

the indication of use of cyclobenzaprine?centrally acting muscle relaxant

the indication of use of tetrahydrozoline:

conjunctival congestion?? in allergies, it's alpha-agonist

the first heart sound corresponds to which of the following points on an atrial pressuretracing?a)a waveb)c wavec)v waved)x descente)y descentc wave

Page 11: 2003 step 1

8/11/2019 2003 step 1

http://slidepdf.com/reader/full/2003-step-1 11/59

© usmile.us 

the second heart sound corresponds to which of the following points on an atrial pressuretracing?a)a waveb)c wavec)v waved)x descente)y descentv wave

S4 corresponds to which of the following points on an atrial pressure tracing?a)a waveb)c wavec)v waved)x descente)y descenta wave

match markers for each measurement:1)evans blue

2)antipyrine3)inulin4)tritiuma)measurement of total body waterb)measurement of plasma compartmentc) measurement of extracellular fluid compartmenta)measurement of total body water: antipyrine, tritiumb)measurement of plasma compartment: evans bluec) measurement of extracellular fluid compartment:inulin

mutation in apoprotein CII is seen in which type of hyperlipidemia?type1:associated with LPL dificiency or mutation in apoCII

Most comonly uesd calcium channel blocker in CHF?a)nifedipineb)amlodipinec)isradipined)diltiazeme)verapamilamlodipine&felodipine

rate limiting step of pyrimidine synthesis?aspartate transcarbamylase

rate-limiting enzyme of cathecholamine synthesis?

thyrosine hydroxylase

side effects of carmustine?Pulmonary toxicity.

 And delayed myelosuppresionPul.Toxicity by Bleomycin, Busulfan,MTX, Carmustine(BBC...MaTriX (Methotrexate)nausea, vomitingtemp. reduction in bone marrow function (anemia)

Page 12: 2003 step 1

8/11/2019 2003 step 1

http://slidepdf.com/reader/full/2003-step-1 12/59

© usmile.us 

what’s the final product of glycolysis?pyruate aerobic,lactate anaerobic

rate-limiting enzyme in glycogenolysis?glycogen phosphorylase

rate-limiting enzyme in gluconeogenesis?Fructose 1-6Bisphosphatase.Stimulated by , ATP,Glucagon, inhibited by Insulin,AMP,Fructose 2-6 Bisphosphate.pyruate carboxylasePEPCK

what enzyme difficiency is seen in heredity fructose intolerance? Aldolase B(Fructose 1P Aldolase),Hereditary F intolerance.

 Aldolase B found in Liver and Kidney.not evident till babay is on mother's milk.

 After Fructose ingestion, Sever Hypoglycemia ( lethargic baby, diaphoresis) and Sevr Lactic Acidosis.Fructose 1 phosphate accumulate in Liver(hepatomegaly,hyperbili,liver damage,hypoglycemia)

and Proximal Tubular Acidosis (Fanconi) in Kidney.Negative Finding...Cataract is not present, sunce fructose is not a subtrate for aldose redutace) oflen.Tx:Eliminate fructose.

the skeletal muscle relaxant that release histamine?Tubocurarine

rate-limiting enzyme in fatty acid synthesis?acetyl-coA carboxylase

 Acetyl CoA carboxylase, needing Biotin, ATP Activated by Insulin,Citrate, High Carb Dieat, Low fat Diet.Inhibited by Glucagon,Free Fatty acids in Blood, High fat diet.

rate-limiting enzyme in HMP shunt?G6PDG6PD.The great enzyme, only rate limiting step in HMP.HMP.

 Also yeilds 2 NADPH. help in Fatty acid synthesis, Keep Glutathione reduced and Oxidativekilling of bacteriain PMN.

BTW, there are other ways to get Ribose 5 Phosphate with out using G6PD, is Tranketolases,and Glycolytic intermediates, F6P,Gleceraldehyde3P are used as intermediate....But they needThiamine . Transketolases are found in RBC for your info.Other thing to remebr is we can differentiate G6PD deficiency from Pyruvate Kinase

deficiency(which give hemolytic anemia too) is that G6PD has heinz bodies , the latter does not.heinz bodies.In mediteranean it is fava beans.In america it is drug induced , infections and African Americans too

the phase of cell cycle affected by cytarabine?Sphase

enzyme difficiency in Tangier’s disease?Familial alpha-Lipoprotein deficiency, absence of apoA

Page 13: 2003 step 1

8/11/2019 2003 step 1

http://slidepdf.com/reader/full/2003-step-1 13/59

© usmile.us 

Low sr. Chol, and HDL.High TAG,VLDL ,Chylomicron.lArge orAnge Tonsils,HepAtomegaly,Neurological deficits.......

enzyme difficiency in Fabry disease?alpha galactosidaseXlinked, accumulation of cermide trihexoside, Renal failure.

enzyme difficiency in Hunter &Hurler diseases?Hunter: Aim for X, Xlinked, iduronase sulfatase, NO corneal clouding and mild MRHurler:Alpha - L iduronase. Corneal clouding and MR.Drawifsm. Not diagnosed in infancy.

drug of choice for cysticercosis?and mechanism of action?praziquantal- increases Ca membrane permeability causing loss of calcium.albendazole alternative treatment

SIADH a side effect of which drug?

a)gancyclovirb)foscarnetc)amantadined)vidarabinevidarabine

the only fatty acid that’s gluconeogenic?propionic acid

mechanism of action of foscarnet?inhibits viral replication by blocking the pyrophosphate binding site of viral DNA polymerase

drug of choice for blastomycosis?

itraconazole choiceamphoB for severe cases

which chemotherapeutic drug considered a cell-cycle nonspecific drug?a)hydroxyureab)etoposidec)metothroxated)busulfanbusulfan:alkylating agents cell-cycle nonspecific

drug of choice for African sleeping sickness:CNS stage?hemolymphatic stage?Melarsaprol,pentamidine

drug of choice for American sleeping sickness?Nifurtimox

drug of choice for Wulchereria bancrofti?Diethylcarbamazine

enzyme blocked by hydroxyurea?ribonucleotide reductase

this apoprotein required for the uptake of LDL in peripheral tissues?

Page 14: 2003 step 1

8/11/2019 2003 step 1

http://slidepdf.com/reader/full/2003-step-1 14/59

© usmile.us 

apoB100

drug that prevents renal toxicity of cisplatin? Amifastine

drug that prevents neutropenia of paclitaxel?Filgrastim

drug that protects against cardiotoxicity of adriamycin?dexrazone

this antiepileptic drug blocks sustained repetitive firing by blocking voltage-dependentNa+channels:a)ethosuximideb)tiagabinec)lamotrigined)topiramatee)vigabatrinlamotriginealso remember that its side effect is rash,estevens-johnson syndrome

this antiepileptic drug is a chemical relative of fructose?a)ethosuximideb)tiagabinec)lamotrigined)topiramatee)vigabatrintopiramate

This antiepileptic drug decreases the efficacy of OC?a)ethosuximideb)tiagabinec)lamotrigine

d)topiramatee)vigabatrinTOPIRAMATE

detachment of epithelial cells from glomerular BM hallmark for this GN?FSGS, FOCAL SEGMENTAL GLOMERULOSCLROSIS

HCV serology should be obtained with diagnosis of this GN?FSGS:Focal Segmental GN

GN associated with atopic history?minimal change disease

subendothelial deposits in which type of GN?MPGN typeI and SLE:typeIV:diffuse proliferative form

estrogen decrease release of this IL from osteoblasts?yes,IL-1,osteoclast activating factoralso i read from UCV that estrogen inhibits IL-6

a sharply demarcated,red velvety lesion of vulve,microscopic examination:large cells withvaculated cytoplasm containing mucin.

Page 15: 2003 step 1

8/11/2019 2003 step 1

http://slidepdf.com/reader/full/2003-step-1 15/59

© usmile.us 

What’s the diagnosis?Paget disease of the vulva, mucin+ cells

a 45yr old man with epididymitis, most common organism responsible?>35yr old: E.coli, pseudomonasless than 35-chlamydia

RFs for vulvar SCC?HPV16,18DM,HTN,obesitysmokingimmunosupression

the most common immature element found in immature ovarian teratoma?Neuroepithelium

OC protects against which organism? gonorrhea or Chlamydia?it's protectice against gonorrhea not chlamydiaOc thinkens cervical mucus, rending it difficult for Gonorrhea to invade the cells.It decreases PID incidence by same mechanism I guess.

which malignancy with hobnail cells?hobnail cells associated with clear cell carcinoma: including: renal cell carcinoma, ductal cellcarcinam (clear cells),clear cell adenocarcinoma of ovary (not related to DEB),clear celladenocarcinoma of vaginaif u want to see hobnail cells,go to:http://radiology.uchc.edu/eAtlas/GYN/1940.htm

cells with Orphan Annie nuclei associated with this cancer?papillary carcinoma of thyroidto see these cells go to :http://pathophysiology.uams.edu/Spring02/pappas/Lecture%2013/sld013.htm

how u detect ferruginous bodies and related disorder?it's associated with asbestosis(more involved lower lobes!)these bodies:yellow-brown rod bodies stain with prossian blueto see ferruginous bodies,go to1) http://www.med.uiuc.edu/PathAtlasf/WWW_images/V4111.html2) http://www.med.uiuc.edu/PathAtlasf/WWW_images/V4363.htmlmost common presntation pleural plaques usually calsifies-bilateral calcified plaques in x-raymain diagnosis is y x-ray in the form of irregular linear opacities mainly in lower lobe.

can also use hrct...but history of exposure and duration when combined with radiological featureshelp in diagnosis

what’s the cofactor for the enzyme that’s deficient in maple-syrup urine disease?

the enzyme defecient:Branched chain alpha keto acid dehydrogenasecofactor:vitB1:thiamine

DNA repair syndrome with defect in DNA ligase?Bloom syndrome

renin changes with these drugs?(increase or decrease?)1:captopril2:clonidine

Page 16: 2003 step 1

8/11/2019 2003 step 1

http://slidepdf.com/reader/full/2003-step-1 16/59

© usmile.us 

3:propranolol4:minoxidilin capto, minoxidil increasein propranolol,clonidine decreasecaptopril inhibits ACE,decrease in AngII levels,feedback increase in renin levelminoxidil a vasodilator,vasodilators increase plasma renin concentration leads to sodium andwater retentionclonidine: alpha-2 agonist that decreases central adrenergic outflow,leads to decrease in vascularperipheral resistance,sodium and water retention,secondary decrease of renin level(that's whyclonidine a good choice for treatment of HTN in patients with renal disease coz it does not reducerenal blood flow,and that's why we usually prescribe clonidine with diuretic!)propranolol:decrease BP leads to Na,water retention,secondary decrease of renin level

a serum sample from a 28yr old female bird-handler suspecting of having psittacosis isevaluated by a complement fixation test.Lysis of erythrocytes happens, what’s ur diagnosis?answer: the patient is not affected!as no Ab present,fixation of complement does not occur and complement lysis the RBCsif this patient had psittacosis,Abs in her serum made a complex with Ags(in test environment)Ag-

 Ab complex fixed complement so no complement was available to lysis RBCs

which one cause both bradycardia and vasodialation?why?a)parasympathetic nerve stimulationb)injection of Achc)bothd)none of theminjection of Achmuscarinic receptors in vascular smooth muscle cells are not inervated,so parasympatheticinnervation only lowers heart rate,but injection of Ach stimulates muscarinic receptors and bothevents happen

this IL transforms macrophages to epitheliod and giant cells?that's IL-4are u sure about IL-2?

IL that’s responsible for conversion of Bcells to plasma cells?IL-5

the only enzyme of TCA cycle that use FAD as a cofactor?succinate dehydrogenase

fanconi anemia at risk for which type of leukemia?

 AML, Acute myeloblastic leukemia

a 65yr old man who recently moved to Phoenix,Arizona,presents with fever,flu-likesymptoms,nonproductive cough and painful red nodules on ant aspect of his lower left leg.a

solitary coin lesion with an egg shell-like cavity is noted in the upper portion of his left lower lobeon Cxray.what’s the diagnosis?coccidiodomycosisPhoenix,Arizona:SW deserts

the effect of clonidine on?1:cardiac output2:peripheral resistance3:rennin activityall decrease

Page 17: 2003 step 1

8/11/2019 2003 step 1

http://slidepdf.com/reader/full/2003-step-1 17/59

© usmile.us 

clonidine:alpha-2 agonist that decreases central adrenergic outflow,leads to decreaseCO,decrease in vascular peripheral resistance,sodium and water retention,secondary decreaseof renin level(that's why clonidine a good choice for treatment of HTN in patients with renaldisease coz it does not reduce renal blood flow,and that's why we usually prescribe clonidine withdiuretic!)

the effect of hydralazine on?1:cardiac output2:peripheral resistance3:rennin activityPR decrease,CO increase,renin increasehydralazine is a vasodilator,vasodilators increase plasma renin concentration leads to sodiumand water retentionas a vasodilator decrease peripheral resistance,also reflex stimulation of the heart that increasesheart contractility and heart rate leads to increase in cardiac output

enzyme deficiency in essential fructosoria and fructosemia?in essentiel fructosuria :fructokinasealdolase B : fructosemia?it 's fructosemia seen in both that is leading to fructosuria due to overload on kidney reabsorption

defenitive diagnosis of fructosemia in a symptomatic infant?liver biopsy to detect aldolase B activity

involvement of which tract responsible for ataxia seen in lateral medullary syndrome?Spinocerebellar

by which mechanism vitB12 defeciency causes functional folic acid deficiency?In the absence of vitB12 methylTHF can’t transfer its methyl group to homocysteine to formmethionine- homocysteine methy transferase enzyme

the main side effect of didanosine,zalcitabine?the main side effect of didanosine is pancreatitis,also dose-limiting toxicity of drug causes painful

peripheral sensorimotor toxicitythe main side effect of zalcitabine is painful peripheral sensorimotor neuropathy,it also can causepancreatitis

autoregulation of blood flow occurs in the circulation of which organs?heart(coronary c.)brain(cerebral c.)

kidney(renal c.)

a motile gr- rod that ferments lactose and forms indole from tryptophan:a)E.colib)proteus

c)pseudomonasd)klebsiellae)enterobacterE.coli

this vitamin used to treat methemoglobinemia?vitC,bcoz it reduces ferric to ferrous iron in the heme molecule

this vitamin is primarily involved in amino acid metabolism?vitB6

Page 18: 2003 step 1

8/11/2019 2003 step 1

http://slidepdf.com/reader/full/2003-step-1 18/59

© usmile.us 

vitamin that acts as a cofactor for conversion of propionyl coA to succinyl coA?yes,2reactions,first biotin for propionyl coA carboxylase(remember that's a cofactor forcarboxylation reactions)that converts it to methylmalonyl coA,and then vitB12 cofactor formethylmalonyl coA mutase that converts methylmalonyl coA to succinyl coA

which cells contain Weibel-palade bodies?endothelial cells

enzyme inhibited by etoposide?topoisomerase2

the location of ketone body synthesis:mitochondria or cytoplasm?Mitochondria

measurement the activity of this enzyme in RBCs used as a clinical marker in vitB1defeciency?Transketolase

vitamin acts as a cofactor for the reaction that converts phosphatidylserine to

phosphatidylethanolamine?the enzyme Phosphotidylserine decarboxylase,i concluded that B1(thiamine)can be a cofactor asit's cofactor for decarboxylation reaction.am i right?

Guthrie test for newborn screening of what disease?PKU

amino acids that essential only in infants and children?arginine,histidine

HMP shunt activity changes in overnight fast?decreased

the change in lung compliance in asthma?compliance increases, elasticity decreasesaccording to BSS CD,it's unchanged!

reduplication of meiotic chromosomes a characteristic of which disorder?a)dermoid cystb)struma ovariic)choriocarcinoma

d)fibromathe answer is a:actually reduplication of maternal meiotic chromosomes:all benign ovarianteratomas or dermoid cysts are of 46,XX karyotype,these tumors r abberent fetuses solely ofmaternal origin arising after the first meiotic division

testosterone decreased in old persons, so u expect less DHT produced, but why BPHcorrelates with old age?with old age, estrogen increased, estrogen promotes expression of receptors for residual DHT,leads to prostate growth, even though testosterone decreased

the effect of lovostatin on anticoagulant effect of warfarin?increased effect

metabolic disease with accumulation of galactocerebroside?

Page 19: 2003 step 1

8/11/2019 2003 step 1

http://slidepdf.com/reader/full/2003-step-1 19/59

© usmile.us 

krabbe's disease

Metabolic disease with deficiency of glucocerebrosidase?gaucher disease

Metabolic disease with deficiency of alpha-galactocerebrosidase A?fabry

metabolic disease with accumulation of glucocerebroside?gaucher disease

the rennin main function in neonates?reNNin like pepsin,it secreted as prorennin and activated by gastric HCl to make rennin,but it’sactivated in higher levels of PH than pepsin,that’s important in neonates that their gastric juice isnot as acidic as adults and its main function is initiation of digestion of milk proteinsdon't mixed it with renin from kidney!

a medical Dr who spends most of his day sitting in front of the computer doing Qbank forstep I,is developing increased girth around his waistline. He wasn’t an overweight as a child orduring his adolescence, his obesity most likely caused by:

a)hyperplasiab)metaplasiac)hypertrophyd)anaplasiahypertrophyin child fat cell increases by hypertrophy&hyperplasiain adult " " " " only hypertrophy

when u see physiologic goiter?puberty and pregnancy

which one is not a characteristic of muscle fibers type II?a)dominant fibers in biceps muscle

b)react to training with hypertrophyc)pale even with ATPase staining at an alkaline PHd)rich in glycogen,poor in mitochondriaC:dark not pale

muscle fibers typeII:fast-twitch muscle in white muscles:like biceps(typeI:slow-twitch in red muscles like posturalmuscles)poor mitochondria,rich in glycogen(opposite seen in typeI)react to traning with hypertrophy(size of typeI unchanged)faster,shorter,more powerful contraction than typeImore enzymes for anaerobic glycolysis than typeIdark ATPase staining at an alkaline PH(typeI:pale)

malakoplakia is most common seen in which organ?malakoplakia:degeneration process in macrophages,most common site:bladder:raised,yellowplaques in bladder mucosa:foamy macrophages that PAS+ and contain Michaelis-Gutmannbodies which r composed of Ca&iron

what’s the difference between pathogenesis of these 2 diseases?Common variable immunodeficiencyBruton agammaglobulinemiadifference in pathogenesis:

Page 20: 2003 step 1

8/11/2019 2003 step 1

http://slidepdf.com/reader/full/2003-step-1 20/59

© usmile.us 

Common variable immunodeficiency:Mature Bcells can’t differentiate to plasma cells,so we haveBcellsBruton agammaglobulinemia:preBcells can’t differentiate to Bcells:lack of Bcells

IL important for mast cell synthesis?IL-3, il-4

citrulline accumulates in deficiency of which enzyme of urea cycle?a)carbamoyl-P synthetaseb)Ornithine transcarbamoylasec)Argininosuccinate synthetased)Argininosuccinate lyasee)Arginasec)Argininosuccinate synthetase

which product of urea cycle forms in mitochondria?a)citrullineb)ornithinec)arginosuccinated)arginine

a)citrulline

which one is associated with secondary adrenal insufficiency?a)K+increased,glucose increased,pigmentation unchangedb)K+increased,glucose unchanged,pigmentation unchangedc)K+increased,glucose decreased,pigmentation increasedd)K+unchanged,glucose decreased,pigmentation decreasede)K+increased,glucose decreased,pigmentation decreasedthe answer is D:K+unchanged,glucose decreased,pigmentation decreasedin adrenal insufficiency :lack of ACTH:so pigmentation decreased,as ACTH has a main control oncortisol,it decreased and results to hypoglycemia,but aldosterone unaffected and K+unchanged

which tumor also called zebra tumor?where u can see zebra bodies?zebra tumor:aucostic neuromazebra bodies:Nieman Pick

what is Pautrier’s microabscess?and associated disease?malignant CD4+T cells in epidermis,MF

what’s Munro’s microabces?and associated disease?collection of neutrophils in epidermis,psoriasis

which Ab acts by misreading of mRNA?

a)erythromycinb)tetracyclinec)streptomycind)chloramphenicolit's streptomycin:aminoglycosides actually

this Ab blocks translocation of the peptide chain from the aminoacyl site to peptidyl site?a)erythromycinb)tetracyclinec)streptomycin

Page 21: 2003 step 1

8/11/2019 2003 step 1

http://slidepdf.com/reader/full/2003-step-1 21/59

© usmile.us 

d)chloramphenicola)erythromycin

in a patient with hypomagnesemia:a)total Ca decreased,ionized Ca unchangedb)total Ca unchanged,ionized Ca decreasedc)both decreasedd)both increasedyes,both decreasedMg is a cofactor for adenylate cyclase,so in hypomagnesemia PTH can’t activate adenylatecyclase and both total and ionized Ca decreased

the location of synthesis of erythropoietin in kidney?endothelial cells of peritubular capillaries in renal cortex

removed during processing of eukaryocytic mRNA?a)CAATb)CCAc)GAATTCd)GU…….AG

e)TATAf)UAGG)polyAh)polyadenylation sequencegood!d:GU…….AGinterons are spliced out during posttranscriptional processing of eukaryocytic mRNA,theseinterons begin at their 5’end with GU and end at their 3’ end with sequence:AG

during posttranscriptional processing of tRNA which one added at the end of tRNA?a)CAATb)CCAc)GAATTCd)GU…….AG

e)TATAf)UAGG)polyAh)polyadenylation sequenceb)the base sequence:CCA

indication of use of domperidone?nausea after radiation therapyalso thiethylperazine another choice to control it

indication of use of domperidone?nausea after radiation therapy

also thiethylperazine another choice to control italso for increasin prolactin secretion for breast feeding purposes

the effect of warfarin on phenytoin level:increased

a person with sickle cell symptoms+splenomegaly+target cells,what’s ur diagnosis?Hgb SC disease

Which coagulating factor used as a marker of tumors?and marker for which tumors?

Page 22: 2003 step 1

8/11/2019 2003 step 1

http://slidepdf.com/reader/full/2003-step-1 22/59

© usmile.us 

factor VIIIangiosarcoma,Kaposi sarcoma

normal flora of nasopharynx? Anaerobic bacteria:peptostreptococcus,fusobacterium,bacteroidsViridans group streptococciCoagulase- staph

 Avitrulant H.influ,N.meningitides

mechanism of action of nystatin?act like amphoBbinds to ergosterol...forms pores in fungal memb

marker of carcinoid tumor?5hydroxyindole acetic acid, also S-100

Giant cells that form by fusion of lymphocytes are seen in which infectious disease?Warthin-Finkeldey cells in measlesMost giant cells are composed of histiocytes, but the WF giant cell iscreated by fusion of lymphocytes

 A certain substance, which is both freely filtered and secreted, is being maximally secreted. As the plasma concentration of the substance increases, the renal clearance: A. decreases and approaches that of inulinB. increases and approaches that of inulinC. increases to the renal plasma flowD. will decrease to zeroE. will remain the same

 A correct, Cos if the conc is still increased it cant be secreted and the excess remains in plasma.Itcant decrease below inulin cos it is secreted.

 A 35-year-old man is referred to the renal clinic for evaluation of proteinuria. He has nocomplaints other than

foamy urine. The following data are obtained from the patient:Inulin clearance 100 mL/minPlasma osmolarity 286 mOsm/LPlasma sodium concentration 140 mEq/LUrine flow 1.44 L / 24 hourUrine osmolarity 205 mOsm/LUrine sodium concentration 100 mEq/LHow much sodium does this patient reabsorb each day?

 A. 14 mEqB. 144 mEqC. 244 mEqD. 20,016 mEq

E. 20,160 mEqabsorbed load= filtered -excreted.filtered=GFRxplasma Na.now here is the mysterygfr=100ml/mt.you have to get in 24 hrs in litres.so that will be .1 L/mt or .1 x 60 x 24 in one day ie144..now therefore filtered load =144x140..exreted load = v x urine sodium conc.=100x1.44=144.therefore absorbed load=144x140-144=20016.hope u got it.

what’s the difference between transvestic fetishism and transsexualism?Transsexuals might cross dress but they do so due to persistent discomfort with their anatomicsex. Transvestic fetishism involves no such discomfort.

Page 23: 2003 step 1

8/11/2019 2003 step 1

http://slidepdf.com/reader/full/2003-step-1 23/59

© usmile.us 

folic acid required for synthesis of which nucleotides?folate involves in the transfer of carbons 2,8 of purine nucleotides(A,G)and 5-methyl group ofthymidine

trisomy with rocker-bottom feet?Both 13, and 18don't forget that it's seen in both trisomy,but polydactyly and midline face defects like cleftpalate.........with trisomy13

metabolic disorders with corneal clouding?Hurler, Fabry, I-cell disease

the eFfect of propranolol on preload?increased preloadpropranolol inhibits beta2 receptors:inhibit vasodilation,and hypotension caused by drug leads toreflex peripheral vasoconstrictionalso hypotention increases Na retention and plasma volume

6 medical students receive test scores of 20,20,40,50,100,30the distribution of this group of scores is:

a)normalb)Gaussianc)skewed to rightd)skewed to lefte)bimodalmode:20median:35mean:43.4mean>median>modeso skewed to right

indication of use of tizanidine?it's structurally related to clonidine, produces muscle relaxation and reduces spasticity by

increasing the presynaptic and postsynaptic inhibition of motor neurons.

this vitamin acts as a component of glutathione reductase?it's riboflavinwhat about vitE?u read about it?i know that acts like antioxidant though

organism that can grow at 6.5%NaCl?Organism that can grow at 8%NaClorganism grows at 6.5%NaCl?enterococcus

Organism grows at 8%NaCl:vibrio.parahemolyticus,vibrio.vulnificus

This drug is NMDA receptor antagonist?

a)LSDb)PCPc)cocained)baclofene)amphetaminePCP

a 5HT1A receptor agonist ?a)buspironeb)sumatriptan

Page 24: 2003 step 1

8/11/2019 2003 step 1

http://slidepdf.com/reader/full/2003-step-1 24/59

© usmile.us 

c)cisapridee)olanzapinebuspirone:5HT1A receptor agonist

a 5HT1D receptor agonist ?a)buspironeb)sumatriptanc)cisapridee)olanzapinesumatriptan:5HT1D receptor agonist

a 5HT4 receptor agonist ?a)buspironeb)sumatriptanc)cisapridee)olanzapinecisapride: 5HT4 receptor agonist

a 5HT2 receptor antagonist ?a)buspirone

b)sumatriptanc)cisapridee)olanzapineolanzapine:5HT2 receptor antagonist

contents of femoral sheath?femoral A&V,femoral branch of genitofemoral nerve,femoral canal(contains lymph nodes)femoralnerve outside the femoral sheath

contents of adductor canal?femoral vessels,saphenous nerve,nerve to vastus medialis

cross over happens in which phase of meiosis?

prophase meiosisI

this element required for DNA polymerization?Mg

defect in which enzyme is seen in Ataxia-Telangectasia? AP endonucleae

the mechanism of action of actinomycinD?

it binds to DNA,inhibits RNA synthesis by blocking movement of RNA polymerase along thetemplate.

what’s the cellular location of assembly of 60s,40s?60S:nucleolus40s:nucleus

which phase of protein synthesis affected by puromycin?it inhibits elongation in both eukaryocytic,prokaryocytics

which phase of protein synthesis affected by sparsomycin?sparsomycin inhibits formation of initiation complex

Page 25: 2003 step 1

8/11/2019 2003 step 1

http://slidepdf.com/reader/full/2003-step-1 25/59

Page 26: 2003 step 1

8/11/2019 2003 step 1

http://slidepdf.com/reader/full/2003-step-1 26/59

© usmile.us 

mechanism of action of paclitaxel?anaphase arrestboth vincristine n paclitaxel are M phase specificbut vincristine binds to tubulin n inhibits polymerisation of microtubule so that microtubule is notformedpaclitaxel hyperstabilises the microtubule so that it canot be broken downpaclitaxel binds to tubulin,but unlike vinca alkaloids promotes polymerization and stabilization ofthe polymer rather than disassembly,so microtubules form but these microtubules aredysfunctional,causing death of the cell:vinca alkaloids binds to tubulin,blocks the ability of tubulinto polymerize to form microtubulescolchicine binds to tubulin causing its depolymerization

reservoir of boleria recurrentis?borrelia recurrentis:reservoir:only humansvector:human body louse

reservoir and vector of Ehrlichia?deer is reservoir(may be from Qbank!)but according to jawets dogs are the main reservoir

this element is a cofactor for the enzyme inhibited by alloporinol?xanthine oxidase, Molybdenum

by decreasing the upper limit of normal of a test,what changes are seen in:1)sensitivity?2)PPV?3)false negative rate?sensitivity:increasedPPV:decreasedfalse negative rate:decreased

the embryonal origin of:1)right ventricle?

2)aortic outflow tract?3)coronary sinus?4)SVC,IVC?according to FA:the embryonal origin of right ventricle:bulbus cordis(in HY:smooth part from the bulbous cordistrabeculated part from the primitive right ventricle)the embryonal origin of aortic outflow tract:bulbus cordisthe embryonal origin of coronary sinus:L horn of sinus venosusthe embryonal origin of SVC,IVC:SVC from cardinal veins,IVC from both vitelline and cardinalveins

cephalosporin can cause serum sickness?

Cefaclor

cisplatin affects which phase of cell cycle?G1,S

organism with bipolar staining?yersinia pestispasteurella multocida

the function of Rb gene as a tumor suppressor gene?

Page 27: 2003 step 1

8/11/2019 2003 step 1

http://slidepdf.com/reader/full/2003-step-1 27/59

© usmile.us 

prevents a cell from entering Sphase when no growth factor presents

the function of p53 gene as a tumor suppressor gene?prevents a cell with damaged DNA from entering Sphase

the embryonal origin of lesser and greter horns of thyroid?lesser:arch2,greater arch3

viral enzyme inhibited by accyclovir?why CMV resistant to this drug?inhibits viral DNA polymerase,CMV resistant to acyclovir coz it lacks a specific viral thymidinekinase

pathogenesis of Zellweger’s disease?it’s due to lack of machinery proteins needed to recognize and import the various peroxisomalenzymesthese patients fail to oxidize very long chain fatty acids

 AA(amino acid)rich in intermediate filaments?Cysteine

hydroxylation of proline and lysine during collagen synthesis occurs in which cellular

organelle?rERHydroxilation of Proline and Lysine in RER, but formation of Triple helix occurs extracellularly,right...?

 Also deficiency of prolyl hydroxylase (in the RER) leads to Scurvy.Deficiency of lysyl oxidase extracellularly leads to Menke's disease.

the main function of:DNA polymerase alpha?beta?teta?gamma?

 Alpha&teta:DNA replicationBeta,epsilon:DNA repairGamma:mitochondrial DNA replication

match these:1)it has a binding site for tRNA2)it binds to mRNA,finds the start codon AUG3)it has peptidyl transferase activity

 A)40SB)60S1)it has a binding site for tRNA 40s2)it binds to mRNA,finds the start codon AUG 40s3)it has peptidyl transferase activity 60s

the location of N-glycosylation?O-glycosylasion?N-glycosylation:addition of sugars to asparagine:begin in rER,complete at golgi

O-glycosylasion:addition of sugars to serine:done in golgi

the location of long-chain fatty acid beta-oxidation?begin in peroxisomes,complete in mitochondria

intermediate filaments in smooth muscle?vascular:vimentin, nonvascular: desmin

enzyme inhibited by miglitol?alpha-glucosidase

Page 28: 2003 step 1

8/11/2019 2003 step 1

http://slidepdf.com/reader/full/2003-step-1 28/59

© usmile.us 

type of epithelial tissue in following organs?a)lining of prostateb)epidydimisc)pulmonary brochiolesd)pigment epithelium of retinaa)lining of prostate:stratified columnarb)epidydimis:pseudocolmunarc)pulmonary brochioles:simple colmunard)pigment epithelium of retina:cuboidal

this antidiabetic drug improves insulin action by activating PPAR-gamma?troglitazone,rezulin

principal neurotransmitter of REM sleep? Acetylcholine.

main defense mechanism used by patients with paranoid personality disorder?Projection

enzyme inhibited by alpha amantin in process of protein synthesis?RNA polymeraseII

sourse of energy for translocation?GTP

melanin,melatonin derived from what AAs?melanin-tyrosinemelatonin-tryptophan(tryptophan........>serotonin..........>melatonin)

treatment of pyruate dehydrogenase deffeciency?increased intake of ketogenic nutrients

 AA becomes essential in:homocystinuria?PKU?cysteine in homocystinuriatyrosine in PKU

this disease a result of defect in degenerative pathway of tyrosinealkaptonuriadefeciency of homogentistic oxidasecharacterized by ochronosis,dark pig of fibrous tissues and cartilage,urine turns black on standing

amylin deposition in pancreatic islets in which type of DM?

DM typeII

what mechanism leads to hyperglycemia in cushing syndrome?hyperglycemia as a result of increased gluconeogenesis and impaired peripheral utilization ofglucose

the mechanism that leads to capillary BM thickening in DM?according to BRS:due to nonenzymatic glycosylation of membrane protein

Page 29: 2003 step 1

8/11/2019 2003 step 1

http://slidepdf.com/reader/full/2003-step-1 29/59

© usmile.us 

the clinical presentation of pancreas involvement in MENI?Z-E syndrome,hyperinsulinism,pancreatic cholera

Donovan bodies best seen with which type of staining?Wright-Giemsa stain

match these:1)angular atrophy predominantly involving typeII fibers2)angular atrophy seen equally involving typeI,II fibers3)target fibers4)fiber-type groupinga)denervation atrophyb)disuse atrophya)denervation atrophy:2)angular atrophy seen equally involving typeI,II fibers3)target fibers4)fiber-type grouping after renervationb)disuse atrophy:1) angular atrophy predominantly involving typeII fibers

main cause of death in Duchenne muscular dystrophy?weakness of respiratory muscles leads to pneumonia,so pneumonia main cause

difference between pathogenesis(genitic basis) of Duchenne and Beker musculardystrophies?Duchenne = x-linked, DNA coding frame-shift --> formation of stop codonBeker = segmental deletions within gene hat do not cause a coding fameshift --> truncateddystrophin protein (less functional)

virus that may have a role in pathogenesis of paget disease?RSV,measles belong to paramyxoviruses family

the change in level of Ca,P,ALP in osteomalacia?

Ca is decreased/normal, P is variable, ALP is increased/normal

the change in level of Ca,P,ALP in osteoporosis?everything is normal....ALP may be decreased

the change in level of Ca,P,ALP in paget?Ca and P are normal.........ALP is increased

vitamin deficiency seen in females on OC use:vitB6

vit required for porphyrin synthesis?

vitB6

vit required for niacin synthesis(from tryptophan):vitB6

parotid gland is not a typical location for this salivary gland tumor?a)pleomorphic adenomab)warthin’s tumorc)mucoepidermoid tumord)adenoid cystic carcinoma

Page 30: 2003 step 1

8/11/2019 2003 step 1

http://slidepdf.com/reader/full/2003-step-1 30/59

Page 31: 2003 step 1

8/11/2019 2003 step 1

http://slidepdf.com/reader/full/2003-step-1 31/59

© usmile.us 

a)hepatocellular jaundice?normal to decreasedb)hemolytic jaundice?increasedc)obstructive jaundice?decreased

change seen in aldosterone level with cirrhosis?increases due to decreased degradation of aldosterone by liver.

match these:1)inflammation of portal triads2)piecemeal&bridging necrosis3)hypergammaglobulinemiaa)chronic active hepatitisb)chronic persistant hepatitisc)chronic autoimmune hepatitis1)inflammation of portal triads: b)chronic persistant hepatitis2)piecemeal&bridging necrosis: a)chronic active hepatitis3)hypergammaglobulinemia: a)chronic autoimmune hepatitis

a 22 yr old woman with turner syndrome has a 2.5cm mass in right adnexa.an abdominalradiography reveals focal areas of calcifications within the mass.the patient most likely has:

a)cyctic teratomab)dysgerminomac)fibromad)Brenner tumord)gonadoblastomagonadoblastoma40%of turners with mosaism,the presence of Y chromosome increases risk of germ celltumors:most common:dysgerminoma,gonadoblastoma,here tumor with calcification sogonadoblastoma

drug that can block placebo response?Naloxane

enzyme inhibited by nalidixic acid?topoisomeraseII:DNAgyrase

by which mechanism vancomycin inhibit bacterial cell wall synthesis?both inhibit synthesis of bacterial cell wall phospholipids and peptidoglycan polymerization

the mechanism of action of actinomycinD?inhibition of RNA polymerase of both eukaryotes and prokaryotes.

which artery is not a part of cerebral arterial circle(of willis)?a)ACAb)MCAc)PCA

d)Post communicating A

e)Ant communicating AMCA

a patient with von-Gierke’s disease starts fasting,what’s the main source of energy for brainafter first 24hr of fasting?in first 24hr fasting glycogenolysis responsible to provide fuel:glucose for brain,after 24hrgluconeogenesis,then gradually ketogenesis.in von Gierke’s glucose-6-phosphatse defecincy,no gluconeogenesis occurs,so in first 24hr

Page 32: 2003 step 1

8/11/2019 2003 step 1

http://slidepdf.com/reader/full/2003-step-1 32/59

© usmile.us 

glycogenolysis again but after first 24hr ketgenesis will be the main mechanism to provide energyfor brain and the answer is ketones

it’s known that niacin limits progression of atherosclerosis,what’s the mechanism?By lowering circulating fibrinogen and increasing circulating tPA

the effect of isoproterenol on:blood pressure?Peripheral vascular resistance?Heart rate?Isoproterenol: stimulates both beta1,2 receptorsIt increases heart rate and force of contraction, increased CO, decreases peripheral vascularresistance by dilatation of skeletal muscles arterioles, also associated with greatly reduce inmean arterial&DBP, but mild increase in systolic BP

what do objective and projective tests mean in beh?objective test based on Qs easily scored:like MMPI,projective test requires subject to interpret theQs:like Rorschach test,TAT

a patient on simvastatin to control hyperlipidemia,now starts to use cholestyramine,what

happens after use of these 2 drugs together?cholestyramine inhibits GI absorption of simvastatin,so the effect of simva decreases and nofurther decrease in LDL levels seen with combination of 2drugs

ECG of a patient who receives digoxin in therapeutic dose range shows:a)prolonged QT intervalb)prolonged PR intervalc)elevation of ST segmentd)wide QRS complexb)prolonged PR intervalusual ECG pattern of patients get digoxin in therapeutic dose shows:prolonged PR interval,STdepression,Twave inversion and occasional biphasia

cutaneus flushing a side effect of niacin,this vasodilatory effect is due to:a)histamine releaseb)NO productionc)PDGF productiond)local PG productiond:local PG responsible,that's why aspirin effective to control flushing

the only family of viruses that are diploid:retroviruses:ssRNA+,diploid:have 2 copies of their genome RNA

virus with double strand segmented RNA?reoviruses:rotavirus belongs to this family

usual site of latency for EBV?B lymphocytes

when bacteria use MDR pump?bacteria actively export drugs using a multidrug resistance pump, MDR pump:imports protons& inexchange exports certain Abs like quinolonesMDR pump is a Multi Drug Resistance pump that is being discovered on more and more bacteria.It is in the cell wall and is used by bacteria to secrete substances that inhibit toxins (antibiotics)

Page 33: 2003 step 1

8/11/2019 2003 step 1

http://slidepdf.com/reader/full/2003-step-1 33/59

© usmile.us 

mechanism of M.TB resistance to ethambutol?mutation in the gene that encodes arabinosyl transferase

mechanism of bacterial resistance to sulfonamides?active export of drug out of cell,reduced affinity of enzyme

to promote lung maturity in premature infants,corticostroids r administered to mother beforedelivery,which corticosteroid is choice and why?bethametasone coz it has the least binding to protein so more free cortico reach to fetus

virulance factors of yersinia pestis?envelope Ag:F1,endotoxin,exotoxin,proteins v,wf1-capsular,v-proten,w w lipoprotein only yersenia has

bacterial exotoxin that's resistant to destroy at 60 degree C?staph enterotoxin

patients with Parkinson on levodopa treatment should avoid consumption of excess of thisvitamin?which vit and why?vitB6, because levodopa converts to dopamine in peripheral tissues that decreases its effect, the

enzyme responsible is dopa decarboxylase that need vitB6 as a cofactor

the indication of use of alprostadilGOOD!it’s PGE1,maintain ductus open preoperatively to correct PDA or some conditions thatthere's a need that shunt be open,treat impotence

the indication of use of echothiophate iodide?yes,long acting irreversible cholinesterase inhibitor topically glaucoma treatment

what effects are seen in bones of patients with homocystinuriaosteoporosis is a radiographic finding in these patients and clinical cases rarely seenalso increased length, decreased thickness of long bonesthey may have:MR, dislocated lens, DVT, stroke, atherosclerosis soon in life

in diff diagnosis: osteogenesis imperfecta like+ marfanoid features= homocystinuria

what’s the drug interaction between NSAIDs &? Lithium?NSAIDs increase renal absorption of lithium

by which mechanism cimetedine potentiates the effect of procainamide?by which mechanism diltiazem potentiates the effect of digoxin?in these 2 drug interactions both act by decrease in renal excretion of second drug

the mechanism of action of eptifibatide and its clinical use?Cyclic heptapeptide,binds to plt glycoprotein IIb/IIIa prevents binding of fibrinogen to plt GPIIb/IIIareceptor,decrease the incidence of thrombotic complications with acute coronary syndromes

what's the main role of telomere?telomere is a nucleotide sequence: in humans GGGTTA at the end of chromosome:allowsreplication of linear DNA to its full length!

the mechanism that leads to fasting hypoglycemia in alcoholism?the excess reduced NADH that result from the metabolism of alcohol converts pyruate tolactate,so pyruate a key substrate for gluconeogenesis not available AND fasting hypoglycemiashould be expected!

Page 34: 2003 step 1

8/11/2019 2003 step 1

http://slidepdf.com/reader/full/2003-step-1 34/59

© usmile.us 

the role of Shine-Delgarno sequence in prokaryocytes?binding site on mRNA for ribosomes

which one is more likely to be a substrate for the enzyme ribonucleotide reductase?a)adenosine monophosphateb)uridine diphosphatec)guanosine triphosphated)adeninee)inosineb, this enzyme required for reduction of ribonucleoside diphosphate to deoxyribonucleosidediphosphate for DNA synthesis, nucleotide must be in diphosphate level to enzyme works

serum anti-nuclear Ab with a rim pattern is seen in what disorder? Anti-dsDNA:SLE

the most indication use of aldesleukin to treat this cancer:recombinant IL-2:RCC

indication of use of levamisole? just read about its indication of use in colon cancer

consider this pathway:methionine…..>A……>S-adenosylhomocysteine…..>B…..>cystathionine…..>Cand match these:1) a major component of alpha-keratin2)it accepts methyl group from vitB12,transfers to methionine3) it’s involved in synthesis of creatine,epinephrine

 A)substnaceAB)substanceBC)substanceCmethionine….>S-adenosylmethionine(A)….>Sadenosylhomocysteine….>homocysteine(B)….>cystathionine….>cysteine(C)S-adenosylmethionine(A):a high energy compound,involved in synthesis of creatine,epinephrine

homocysteine(B):accepts methyl group from vitB12,transfers to methioninecysteine(C):sulfur containing AA, a major component of alpha-keratin

the mechanism of action of drug used topically to treat condyloma acuminatum?what i know:podophylin contain cytotoxic agents that prevent the formation of the mitoticspindle&mitosisINF for recurrent casespls tell me ur source about the answer

gleomerolonephritis with autoAb against C3bBb?MPGNII,C3bBb or C3convertase or C3nephritic factor

silicosis a RF for which pulmonary disease?

Tuberculosis

a 65yr old man has the following Weber and Rinne test results:Weber lateralizes to left ear

In Rinne test:air conduction>bone conduction in both earsWhat’s the diagnosis?a)presbycosis:right earb)presbycosis:left earc)otosclerosis:right ear

Page 35: 2003 step 1

8/11/2019 2003 step 1

http://slidepdf.com/reader/full/2003-step-1 35/59

© usmile.us 

d)otosclerosis:left eara)presbycosis: right earin presbycosis or sensorineural hearing defect&normal condition,air conduction longer than boneconduction(in conductive defect or otosclerosis:bone conduction longer)so choose betweennormal one and presbycosisas weber lateralizes:it's not normal!it's presbycosis!well,now!which ear?as weber lateralizes to left ear:it's normal ear:remember in presbycosisweber lateralizes to normal one,so the answer is right presbycosis(in conduction defect weberlateralizes to abnormal ear!)

a patient with COPD:blue bloater type is vomiting,what changes are seen inPH,PaCO2,HCO3-?metabolic alkalosis and resp acidosis.PH may be normal if vomiting is prolonged enough.PACO2 highHCO3 is high

1:toxic metabolite of acetaminophen that’s toxic to liver?2:What’s the antidote for acetaminophen toxicity?and the mechanism of action?

 Acetaminophen is primarily eliminated by conjugation with glucoronide and sulfate,but a minor

metabolite:N-acetyl-benzoquinone toxic to liver,this metabolite accumulates after acute overdoseof acetaminophen coz of depletion of sulfhydryl groups required to degrade it,N-acetylcysteineprovides a source of sulfhydryl groups to inactivate this toxic metabolite

what r neurofiblliary tangles that seen in Alzheimer’s disease?a)microfilamentsb)intermediate filamentsc)microtubulesd)microtubules associated-proteinsyes!d)microtubules associated-proteins:tau protein

brain tumor with perivascular pseudorossets?

actually i read about both:epandymoma in kaplan,both in BRS,so all correct,if u find anothersource about it let me know!

the only hypothalamic hormone that’s not a peptide?dopamine:catecholamine

location of carbamoyl phosphate I,II?cps 1 in urea synthesis in mitochondria .ii in pyrimidine synthesis in cytoplasm

which hormone of OC responsible for microglandular hyperplasia of endocervix?Progestrone

acid-base disorder seen in a pregnant woman with excessive vomiting?

respiratory alkalosis due to hyperventilation seen in pregnantsmetabolic alkalosis due to vomiting

what does desmoplasia mean and seen most in which tumors?

adenocarcinoma often ass with desmoplasia:tumor-induced proliferation of non-neoplastic fibrousconnective tissues,particularly with adenocarcinoma of breast,pancreas,prostate

hypoglycemia as a paraneoplastic syndrome most seen in which cancers?hepatocellular carcinomas ,mesotheliomas and some sarcomas

Page 36: 2003 step 1

8/11/2019 2003 step 1

http://slidepdf.com/reader/full/2003-step-1 36/59

© usmile.us 

neoplasm associated with gene amplification?N-myc:neuroblastomac-neu:breast cancerc-erb B2:breast cancer

in CO poisoning neuronal necrosis is most seen in which parts of brain?basal ganglia,lenticular nuclei,cortical gray areas

histone synthesis occurs in which phase of cell cycleSphase

what’s the mechanism of edema formation in left-sided heart failure&R-sidedHF?Left-sided heart failure:reduction in renal perfusion that activates renin-ang systemRight-sided heart failure:renal hypoxiainfact fluid retention&peripheral edema more prominent in R-sided HF but L-sided hf can causeby decreasing renal perfusion

oncogenes erbB1,erbB2 related to which type of cancers?erbB1:SCC lung

erbB2:breast,ovarian,lung,stomach

blood supply of parathyroid glands?inf thyroid artery

enzyme inhibited by lead,responsible for hypochromic microcytic anemia:ferrochetalase and ala dehydrase.

most common arteries involved in Monckeberg’s arteriosclerosis?radial,ulnar

toxic substances that have a role in angiosarcoma formation?arsenic,thorotrast,polyvinyl chloride

by which mechanism hyperparathyroidism leads to HTN?activation of rennin-ang system

enzyme inhibited by sildenafil?cGMP phosphodiesterase type5

factors required for pyruate carboxylase to be activated?biotin,Mg,Mn,acetyl coA

 ATP synthesis occurs in which phase of cell cycle?G2

what’s the main function of Chaperone proteins?they aid in the importation of proteins into the mitochondria: in unfolded state during importationthey're required for correct folding

SnRNPs:

enzyme responsible for their synthesis?their main role?related to which autoimmune disease?synth by rna pol2

Page 37: 2003 step 1

8/11/2019 2003 step 1

http://slidepdf.com/reader/full/2003-step-1 37/59

© usmile.us 

r spliceosomes invoved in REMOVING ITRONS FROM mrnaauto immune diseases r SLE , deramatomyosistis

what’s mechanism of resistance of cancer cells to methotroxate?amplification of DHF reductase gene

in this phase of mitosis:nuclear envelope distruped,nucleolus disappears: A)prophaseB)prometaphaseC)metaphaseD)anaphaseE)telophaseprometaphase

chromosomes isolated for karyotype in this phase? A)prophaseB)prometaphaseC)metaphaseD)anaphaseE)telophase

metaphase

cyclins inactivated by protein degradation during this phase? A)prophaseB)prometaphaseC)metaphaseD)anaphaseE)telophaseanaphase

hormone deficiency seen in mutation of a gene that encodes for Pit-1:a transcription factor?GH,prolactin,TSHPit-1 is a homedomain protein that's required for transcription of these hormones

what kind of mutation in mtDNA occurs in Leber’s heredity optic neuropathy?missense mutation that changes Arg to his

match these:a)inhibits release of glycine through presynaptic bindingb)blocks inhibitory output by binding glycine receptorc)blocks Ach receptor by binding irreversibly to nicotinic receptorsd)inhibits Ach release at NMJe)presynaptic binding causes excessive release of Ach1)alpha-bungarotoxin2)tetanus toxin3)black widow spider

4)botulinum toxin5)scorpion venom6)strychninea:2,b:6,c:1,d:4,e:3,5alpha-bungarotoxin:blocks Ach receptor by binding irreversibly to nicotinic receptorstetanus toxin:inhibits release of glycine through presynaptic binding

black widow spider&scorpion venom:presynaptic binding causes excessive release of Achbotulinum toxin:inhibits Ach release at NMJstrychnine:blocks inhibitory output by binding glycine receptor

Page 38: 2003 step 1

8/11/2019 2003 step 1

http://slidepdf.com/reader/full/2003-step-1 38/59

Page 39: 2003 step 1

8/11/2019 2003 step 1

http://slidepdf.com/reader/full/2003-step-1 39/59

© usmile.us 

b)pulseless diseasec)Rendu-osler-Weber syndromed)VHL diseasee)Churg-Straussf)wegnerc

population most at risk for eosinophilic granoluma?former smokers

atypical pneumonia in pet bird owners,that associated with splenomegaly & bradycardia:psittacosis

what percentage of CO is shunted in a normal person?2%

this type of pneumoconiosis leads to formation of non-caseating granulomas andhistologically identical to sarcoidosis?Berylliosis

high salt diet RF for which cancer?Stomach

the boundaries of the epiploic foramen of Winslow?Hepatoduodenal ligament:antCaudate lobe:supDuodenum:infIVC:post

in Hartnup’s disease defect in absorption of which kind of AAs? and clinical presentationmost due to which AA deficiency?due to natural AA defeciency!(essential neutral)tryptophan def responsible for symptoms

tryptophandiarhoeadermatitisdementiamnemonic = d3 cos of b3) ie vit b3 niacin deficiency gives this picture.while in hartnup disease the precursor aa of niacin (tryptophan) is not absorbed properly leadingto consequent niacin deficiency.

location of absorption of folate,iron,Ca,Mg?all duodenum

GI location of absorption of fat soluble vitamins?what about water soluble vitamins?fat soluble vitamins:duodenum,jejunum

water soluble vitamins:jejunum,ileum

celiac disease:HLA associated?RF for which cancers?HLA-B8,DQW2T-cell lymphoma,GI,breast cancer

glucagon resistant to suppression in what kind of DM?NIDDM:type2

Page 40: 2003 step 1

8/11/2019 2003 step 1

http://slidepdf.com/reader/full/2003-step-1 40/59

© usmile.us 

the main role of vif gene in HIV?cell to cell transmittion

3 disorders with café au lait spots:2 more?!i read about these 3:NF-1,Peutz-Jeghers syndrome,fibrous dysplasia

nerve lesion leads to:medial winging of scapula?lateral winging of scapula?medial winging -long thoracic nervelateral winging-spinal accessory for trapezius

bounderies of snuffbox in dorsal&palmar sides?Dorsal:extensor pollicis longusPalmar:extensor pollicis brevis,abductor pollicis longus

contents of inguinal canal?illioinguinal nerve, round ligament in femalesillioinguinal nerve, spermatic cord(ductus deferens,testicular artery,pampiniform plexus)in males

change in level of LAP in PNH,CML, polycythemia vera?cml n pnh-lowpolycythemia vera-elevated

Job’s syndrome is an example of:a)B cell deficiencyb)T cell defecincyc)combined B,T cell deficiencyd)phagocyte deficiencyphagocyte deficiency

hyper-IgM syndrome an example of:

a)B cell deficiencyb)T cell defecincyc)combined B,T cell deficiencyd)phagocyte deficiencyb)T cell defecincy

what’s the main defect in Job’s syndrome?Immune deficiecne secondary to deficient T cell production of gamma interferons, a cytokinewhitch enhances granulocytic chemotaxism and suppress IgE production , si children with thisdisorder present with high IgE level and prone to recurrent infections.

clotting factor inhibited by alpha-1-Antitrypsin?FactorXIa

the only clotting factor that can’t be inhibited by alpha-2-macroglobulin?factorXIII coz is not a serine proteaseall serine protease clotting factor can inhibited by alpha-2 microglobulin

the only RNA virus that assembled in nucleus?it's influenzaretroviruses assembled in cytoplasm

Page 41: 2003 step 1

8/11/2019 2003 step 1

http://slidepdf.com/reader/full/2003-step-1 41/59

Page 42: 2003 step 1

8/11/2019 2003 step 1

http://slidepdf.com/reader/full/2003-step-1 42/59

© usmile.us 

a culture of bacteria not resistant to tetracycline develops an infection from a virus that’sderived from the lysis of tetracycline-resistant bacteria.most of the bacterial progeny of the originlculture is found to have become resistant to tetracycline,what phenomen has occurred?a)conjugationb)recombinationc)transductiond)transformationtransduction:means transfer of a portion of DNA from one bacterium to the chromosome ofanother bacterium by means of a viral infection

which of the following AAs most compatible with an alpha-helical structure?a)tryptophanb)alaninec)prolined)cysteinee)lysinealanine

the effect of cycloheximide on protein synthesis in eukaryocytes similar to the action ofwhich ab in prokaryocytes?

a)tetracyclineb)streptomycinc)erythromycind)chloramphenicolerythromycin both inhibit elongation

enzyme inhibited by alpha-amanitin?RNA polymeraseII

genetic code is degenerate.what does it mean?more than one codon may exist for a single AA

a potent inhibitor of protein synthesis that acts as an analogue of aminoacyl-tRNA is:

a)mitomycinCb)streptomycinc)nalidixic acidd)rifampicine)puromycinpuromycin

which of the following enzymes is common to both glycolysis&gluconeogenesis?a)pyruvate kinaseb)pyruvate carboxylasec)hexokinased)phosphoglycerate kinasee)fructose 1,6 biphosphatase

d. the only reversible kinase in glycolysis

which one of the following activities is simultaneously stimulated by epinephrine in muscle&inhibited by epinephrine in liver?a) fatty acid oxidationb)glycogenolysisc)cAMP synthesisd)glycolysise)activation of phosphorylated! as it's the only source of energy in muscle, but its inhibition prevent futile cycle in liver

Page 43: 2003 step 1

8/11/2019 2003 step 1

http://slidepdf.com/reader/full/2003-step-1 43/59

© usmile.us 

which of the following events occurs during formation of phosphoenolpyruvate from pyruvateduring gluconeogenesis?a) CO2 is consumedb)inorganic phosphate is consumedc)acetyl coA is utilizedd)ATP is generatede)GTP is generateda:CO2pyruvate+ATP+CO2------->oxaloacetate+ADP+Pioxaloacetate+GTP------->phosphoenolpyruvate+GDP+CO2

citrate has a positive allosteric effect on which of the following enzymes?a)pyruvate kinaseb)acetyl coA carboxylasec)phosphofructokinased)fatty acid synthetasee)enolaseacetyl coa carboxylase

which one of the following tissues can metabolize glucose, fatty acid, and ketone bodies for ATP production?a)liverb)musclec)braind)RBCsb:muscle!muscle cells r the only cells that r capable of utilizing all energysourcesavaible:glucose,FAs,during fasting keton bodies.RBCs can’t utilize FAs,keton bodies,since mitochondria required for their metabolism.FAs can’t cross BBB,so brain can use glucose&keton bodies.Hepatocytes ar the site of keton bodies production, but the mitochondrial enzyme necessary for

utilization of ketone bodies is not present in hepatocytes.

which of the following enzymes activated by cAMP during fasting?a)lipoprotein lipaseb)acetyl coA carboxylasec)pancreatic lipased)carnitine acyltransferaseIe)diacylglycerol lipasef)hormone-sensitive lipasef: another name: TG lipase: activated by glucagons, NE, epinephrine

when the liver is actively synthesizing fatty acids, a concomitant decrease in beta oxidationof fatty acids is due to:

a)inhibition of a translocation between cellular compartmentsb)inhibition by an end productc)activation of an enzymed)decrease in adipocyte lipolysisa. inhibition of the translocation(malonyl coA inhibit carnitin acyltransferase I--> inhibit carnitinshuttle)

feedback inhibition of pyrimidine nucleotide synthesis can occur by which of the followingmeans?a)CTP allosteric effects

Page 44: 2003 step 1

8/11/2019 2003 step 1

http://slidepdf.com/reader/full/2003-step-1 44/59

© usmile.us 

b)UMP competitive inhibition

c)TTP allosteric effectsd)none of aboveafirst enzyme:CAP-synthetaseII feedback inhibited by UMP through allosteric effects on enzymestructure,not competitive inhibitionsecond enzyme:aspartate transcarbamoylase inhibited by CTP

a woman with cystic fibrosis marries her first cousin.what’s the risk that their first child willhave cystic fibrosis?a)1/2b)1/4c)1/8d)1/16e)1/32pls explain the answeryes!1/8woman aaman Aa(risk of being carrier:1/4)

aa+Aa:aa,aa,Aa,Aaso:risk of disease:1/21/4*1/2=1/8man Aa,woman aa:risk of child to be affected:1/2but risk of man(cousin)to be affected is 1/4why?coz as one of their common grandparents is a carrier,1/2 chance that one of his parents bea carrier and 1/2 chance of inheritiing this carrier status from parents,so 1/2*1/2=1/4:risk of cousinto be carrier:1/41/2*1/4=1/8

if parents with 3 affected children have a higher recurrence risk than parents with 2 affectedchildren, the disease is likely to exhibit:a)AD inheritance

b)AR inheritancec)X-linked recessive inheritanced)X-linked dominant inheritancee)multifactorial determination inheritance

polycyctic kidney disease is a significant case of renal failure that presents from earlyinfancy to adulthood.early-onset cases tend to affect one family member or siblings,whileadultonsetcases often show a vertical pattern in the pedigree.which of the following offers the bestexplantation of these facts?a)pleiotropyb)allelic heterogeneityc)locus heterogeneity

d)multifactorial inheritancee)variable expressivityc)locus heterogeneitypolycystic…..an example of genetic heterogeneity:diff mutations may cause similarphenotypes.furthur divided into:allelic&nonallelic(locus)heterogeneity:allelic H:diff mutatons at the SAME locus that both result in similar disease:marfanlocus(nonallelic) H:mutations at DIFFERENT loci,yet the phenotype similar

a man whose brother has cystic fibrosis wants to know his risk of having an affectedchild.the prevalence of CF is 1 in 1600 indivisuals.the risk in this case is:

Page 45: 2003 step 1

8/11/2019 2003 step 1

http://slidepdf.com/reader/full/2003-step-1 45/59

© usmile.us 

a)1/8b)1/16c)1/60

d)1/120e)1/256man should be Aa(chance of being carrier:2/3)woman should be carrier:Aa(the only possibility tohave a child with CF)chance of being carrier for woman in this population is:2pq=2*1/40=1/201/4 risk for child to be affectedso:2/3*1/20*1/4=1/120

the most efficient&accurate method for detecting the deletion in Prader-Willi syndrome?fluorescent insitu hybridization

the difference between allelic &nonallelic heterogeneity?allelic H:diff mutatons at the SAME locus that both result in similar disease:marfanlocus(nonallelic) H:mutations at DIFFERENT loci,yet the phenotype similar:kidney polycycticthat's AR in infants,AD in adults

a 3 month-old boy presents with poor feeding and growth,hypotonia,lactic academia,and nild

acidosis,the ratio of pyruate to lactate in serum is elevated,and there is decreased conversion ofpyruate to acetyl coA in fibroblasts,which of the following compounds might be considered fortherapy?a)pyridoxineb)thiaminec)free fatty acidsd)biotine)ascorbic acidan elevation of pyruate & deficiency of acetyl coA suggest a deficiency of pyruatedehydrogenase,requires thiamine:b

a compound normally used to conjugate bile acids is:a)acetate

b)glucuronic acidc)glutathioned)sulfatee)glycineit's glycineglucoronic acid for billirubin

which of the following compounds serves as a primary link between the citric acidcycle&urea cycle?a)malateb)succinatec)fumarated)citrate

e)isocitratefumarate

which of the following is appropriate for a patient with renal failure?a)high carbohydrate dietb)high protein dietc)low fat dietd)high fiber dieta,diet high in carbohydrate and fats spares glucose use and inhibits gluconeogenesis,preventingprotein catabolism&nitrogen production

Page 46: 2003 step 1

8/11/2019 2003 step 1

http://slidepdf.com/reader/full/2003-step-1 46/59

© usmile.us 

 A child with Tay-Sachs disease.what’s the risk that the grandmother of this child is a carrier?a)100%b)67%

c)50%d)25%e)virtually 0pls explain the answerc. 50%. the disease is autosomic recessive so the child is aahis parents must be both Aa but the father or the mother have inherited it from their parents eitherform the granfather or the granmother who are also Aa there is thus 50% of risk that thegranmother has been a carrier

a physician diagnoses genital herpes in a 16yr old male high school student,prior totreatment him,the physician should:a)notify his parentsb)get permission from his parentsc)notify his sexual partnersd)recommend that he tell his sexual partners

e)notify the appropriate state agencyd)recommend that he tell his sexual partners

why we see hypercalcemia in sarcoidosis?epithiloid macrophages convert vit D into active form....vit D causes increased Ca reabsorptionfrom GI and bone

which irreversible enzyme of gluconeogenesis is located in mitochondria?a)pyruvate carboxylaseb)PEP carboxykinasec)fructose 1,6-biphosphatased)glucose-6-phosphatasea)pyruate carboxylase

shuttle that responsible for fatty acid entery into mitochondria:shuttle that responsible for fatty acid entery into cytosol:into mito is carnittineinto cyto is citrate

in electron transport chain which enzyme inhibited by oligomycin?F0 of ATP ase

the effect of CAMP&Ca++on glycogenesis and glycogenolysis pathways?inhibition oractivationthe same for Ca,in summary Ca,cAMP effect on glycogenolysis activation,on glycogenesisinhibition

the effect of alanine on pyruate kinase in glycolysis pathway?inhibitor or activator?Inhibition

match these:1:delivers dietary TGs to peripheral tissues,secreted by intestinal epithelial cells2:delivers hepatic TGs to peripheral tissues,secreted by liver3:secreted from both liver&intestinea)HDLb)LDL

Page 47: 2003 step 1

8/11/2019 2003 step 1

http://slidepdf.com/reader/full/2003-step-1 47/59

© usmile.us 

c)VLDLd)chylomicron1:delivers dietary TGs to peripheral tissues,secreted by intestinal epithelial cells(CHYLOMICRONS)2:delivers hepatic TGs to peripheral tissues,secreted by liver (VLDL)

3:secreted from both liver&intestine (HDL)

vitamin deficiencies seen with INH?BOTH B3&B6:niacin&pyridoxine

angular stomatitis,cheilosis,corneal vascularization.which vitamin deficiency?B2

what’s the storage form of vitaminD?25-OH D3

the effect of mineral oil intake on fat soluble-vitamins absorption?decreased absorption of fat soluable vit.

a 29yr old male patient with psoriasis on his hands&arms asks his doctor how to deal withthe reaction of people when they see the rash.the doctor’s best response is:a)’’act like nothing is wrong’’b)’’wear long-sleeved shirts’’c)’’tell people that it’s not contagious’’d)’’stay at home as much as possible’’e)’’tell people that it’s not their business’’c)’’tell people that it’s not contagious’’

an 8yr old child with normal intelligence reads,communicates well,and gets along well withthe other children in school.however he often argues with the teacher.his parents tell the doctorthat he often seems angry toward them&rarely follows their rules,the best description for this childbehavior:

a)normalb)ADHDc)autistic disorderd)oppositional defiant disordere)conduct disorderyes,d)oppositional defiant disorderas he has good relationship with other children it's not conduct disorder

the ethnic group with the longest life expectancy is:a)asian Americansb)African Americansc)white Americansd)native Americans

yes,asian Americansthen african americans.....then white/native Americans

match DOC for each organism:a)albendazoleb)diethylcarbamazinec)ivermectind)niridazolee)praziquantelf)pyrantel pamoate

Page 48: 2003 step 1

8/11/2019 2003 step 1

http://slidepdf.com/reader/full/2003-step-1 48/59

© usmile.us 

g)thiabendazole1:clonorchis sinensis2:toxacara canis3:Loa Loa4:ancylostoma duodenale5:strongyloides stercoralis

6:onchocerca volvulus7:dracunculus medinensis8:echinococcus granulosus9:paragonimus westermaniclonorchis sinensis- praziquantal2:toxacara canis-DEC3:Loa Loa-DEC4:ancylostoma duodenale-pyrantal pamoate5:strongyloides stercoralis-thiabendazole6:onchocerca volvulus-ivermectin7:dracunculus medinensis-niridazole8:echinococcus granulosus-albendazole9:paragonimus westermani-praziquantal

Frei test+ used for dignosis which organism?LGV:CL.trachomatisbe careful!it's LGV not granolum inguinale

why aminoglycosides r ineffective against anaerobes?why gr- bugs resistant to penicillinG?why aminoglycosides r ineffective against anaerobes?these abs require O2 to eneter into cell wallwhy gr- bugs resistant to penicillinG?the gr- outer membrane layer inhibits entry of penicillinG andvancomycin

4 bacteria with beta hemolysis?staph aureus

strep pyogenstrep agalactiaelisteria monocytogenabout strep fecalis,not heard tnxabout cl.perferingens:i read about double hemolysis,is it beta hemolysis?

4 facultative intracellular bacteria?yersiniabrucellafrancisellalisteriamycobacterium.TB:facultative,M.leprae:obligate intracellular parasitei'm not sure about legionella,is it facultative?

main reservoir of yersinia pestis?rodents,esp.prairie dogs

viral infection with VDRL false+?bacterial infection with VDRL false+?EBV, hepatitisleprosy

RNA viruses with circular viral genome?

Page 49: 2003 step 1

8/11/2019 2003 step 1

http://slidepdf.com/reader/full/2003-step-1 49/59

© usmile.us 

DNA viruses with circular viral genome?HDV with circular RNA?pls confirm, yes rna genome of HDV is circular,i checked it with

 jawets,tnxRNA circular = Arena and BunyaDNA circular = Papovo and Hepadna (partial circular)

UTI bug with metallic sheen colonies on EMB agar?

E.coli

the mechanism of action of dalfopristin?PREVENTS AMINOACYL TRNA FROM BINDING TO "A" SITE,AND DISSOCIATE THETERNARY COMPLEX.LINEZOLID PREVENTS FORMATION OF THE TERNARY COMPLEXtargets 50s ribosome and inhibits elongation.

which statement is not correct about aztreonam?a)it inhibits cell wall synthesisb)it’s synergic with aminoglycosidesc)it has no activity against anaerobes

d)it has no activity against gr+e)it has no activity against pseudomonasf)it’s safe for patients who r allergic to penicillinit inhibits cell wall synthesis,synergic with aminoglycosides,has no activity againstanaerobes&gr+,effective against pseudomonas.it’s safe for patients who r allergic to penicillin

which statement is correct about imipenem?a)it’s effective against pseudomonas strainsb)it has no activity against anaerobesc)it has no activity against enterobacterd)CNS toxicity seen at high plasma levels

D is correct,infact:it’s just effective against pseudomonas aueroginosa,not other pseudo strainsalso effective against anaerobes& drug of choice for enterobacterCNS toxicity seen at high plasma levels

which one is not a side effect of vancomycin?a)ototoxicityb)nephrotoxicityc)neurotoxicityd)thrombophlebitisC. neurotoxicity

in treatment of glaucoma match each drug with its appropriate effect:

a)epinephrineb)carteololc)echothiophated)dorzolamidee)latanoprostf)brimonidineg)carbachol1:increase outflow of aqueous humor2:decrease aqueous humor secretion3:decrease aqueous humor production

Page 50: 2003 step 1

8/11/2019 2003 step 1

http://slidepdf.com/reader/full/2003-step-1 50/59

Page 51: 2003 step 1

8/11/2019 2003 step 1

http://slidepdf.com/reader/full/2003-step-1 51/59

Page 52: 2003 step 1

8/11/2019 2003 step 1

http://slidepdf.com/reader/full/2003-step-1 52/59

© usmile.us 

HT2 and 5-HT3 antagonist. In addition, mirtazapine has only a weak affinity for 5-HT1 receptorsand has very weak muscarinic anticholinergic and histamine (H1) antagonist propertiessourse intalpha2,5HT2

a drug with a half-life of 12hr is administered by continunous IV infusion.how long will it takefor the drug to reach 90%of its final steady-state level?a)18hrb)24hr

c)30hrd)40hre)90hrafter3.3 half lives,so it's D:40hr

which of the following most closely describes the clearance rate of a drug that’s infused at arate of 4mg/min and produces a steady-state concentration of 6mg/dl in the plasma?a)67ml/minb)132ml/minc)300 ml/min

d)667ml/mine)1200ml/minat steady state:infusion rate=excretion rate=4mg/minexcretion rate=Cl*plasma concentration4=Cl*0.006 so Cl=667

the antimicrobial drug,tetracycline is found to be therapeutically effective when 250mg ofdrug r present in the body.the t1/2 of tetracycline is 8hr.what’s the correct rate of infusion?a)7mg/hrb)12mg/hrc)22mg/hrd)37mg/hre)45mg/hr

r= ke xvd x c r is the rate of infusionke =o.7/t1/2 =0.086ke is the constant rate of drug elimination from the body250mg is the amount of drug inthe body which is equal to vd xcbecause the concentration of a drug is the dose on volume of distributionc=D/vdo vd x c equal to Dso D=vdxc =250mgso the rate of loss of tetracyclineper hour is 8.6% of whatever drug is inthe body as we have250mg so the rate of loss is 250x8.6/100=21.5 mg/hrso the answer is C

which of the following results in a doubling of the steady-state concentration of a drug?

a)doubling the rate of infusionb)doubling the loading dosec)doubling the concentration of infused drugd)both a&cd, dont agree hereif you double both you wont double the steady state conmcentration but you will quadruple it

 A 2-year-old child with leukemia develops nephrotic syndrome.Light microscopic studies are normal. Electron microscopic studiesdemonstrate fusion of epithelial foot processes. The current hypothesis

Page 53: 2003 step 1

8/11/2019 2003 step 1

http://slidepdf.com/reader/full/2003-step-1 53/59

© usmile.us 

for the pathogenesis of this change is that it is secondary to whichof the following?

 A. Consumption of complement factorsB. IgG directed against basement membraneC. Immune complex depositionD. Lymphokine production by T cellsE. Mesangial IgA depositionMCD:answer is d826.

HbNl: AAGUAUCACUAAGCUCGCHbCr: AAGAGUAUCACUAAGCUCGCUUUC ... UAU UAAHemoglobin is isolated from the erythrocytes of a young child with anemia.Hemoglobin electrophoresis reveals the presence of an unstable hemoglobin,known as hemoglobin Cranston (HbCr), containing an abnormal b-globin chain.The normal sequence of the b-globin gene (HbNl) and the sequence of theHbCr b-chain are presented above. Which of the following would accountfor the development of HbCr?

 A. A frameshift mutation resulted in the deletion of several amino acid residues in the b-chainB. A mutation in the stop codon resulted in elongation of the b-chain

C. A point mutation resulted in the insertion of a stop codon in the b-chainD. A two base pair addition resulted in the elimination of a stop codon in the b-chainE. A two base pair deletion resulted in truncation of the b-chainyes,d

 A 50-year-old physician is recovering from aseptic meningitis that began two weeks ago. Heappears to have lost considerable cognitive function, and he says he will not go on living if hiscognitive ability remains compromised. To demonstrate to him that recovery is occurring, serialevaluations with which of the following psychological assessments is indicated?

 A. Halstead-Reitan BatteryB. Stanford Binet Intelligence TestC. Vineland Adaptive Behavior ScaleD. Wechsler Adult Intelligence Scale

E. Wide Range Achievement Testa

 A boy with Duchenne muscular dystrophy (DMD) was born to parents with no family historyof the disease. The most likely explanation for this occurrence is

 A. a CGG expansion that resulted in the disruption of the promoter of the dystrophin geneB. infidelityC. a point mutation in the dystrophin geneD. a recombination event in the dystrophin gene that gave rise to a frameshift mutation leading toan untranslatable mRNAE. a translocation that resulted in the disruption of the dystrophin geneanswer is d, parents not affected!

 A 48-year-old white female secretary presents with progressive difficulty typing over the pastmonth. She also notes that her hands begin to feel numb and weak after typing for long periods oftime. Upon testing, which of the following deficits would be predicted?

 A. Difficulty in abducting the fifth fingerB. Difficulty in adducting the thumbC. Difficulty in flexing digits two and three at themetacarpophalangeal jointsD. Loss of sensation over the lateral half of the dorsumof the handE. Loss of sensation over the lateral half of the palm

Page 54: 2003 step 1

8/11/2019 2003 step 1

http://slidepdf.com/reader/full/2003-step-1 54/59

© usmile.us 

F. Loss of sensation over the medial half of the dorsumof the handG. Loss of sensation over the medial half of the palm

 justCsensation over the lateral half of the palm not affected as it's inervated by superficial branch ofmedian nerve that not involved in carpal tunnel syndrome

 A patient develops a form of lung cancer that spreads to occlude the thoracic duct. Edemainvolving which of the following sites might be expected as a potential complication?

 A. Entire left side and right leg

B. Entire right side and left legC. Left arm onlyD. Right arm and right half of head onlyE. Right arm onlya

adverese effects of interferon therapy is most severe in which organ?a)liverb)heart

c)kidneyd)lungheart,CHF

the indication of use of IFN alpha,beta,gamma? Alpha:chronic hepatitis B,CGenital warts by papillomavirusHairy cell leukemiaKaposi sarcomaBeta:Multiple SclerosisGamma:CGD

an adenosine analogue that’s active against all members of the herpesvirus group that

infects humans?a)amantadineb)ribavirinc)vidarabined)ganciclovire)acyclovirno answer

 A patient with Hodgkin disease dignosis,the doctor warned him to avoid foods that containtyramine during chemotherapy, why?No answer

match each class of antiarythmetic drugs with its appropriate effect:

a)slows phase0 depolarization:b)shortens phase3 repolarization:c)prolongs phase 3 repolarization:d)suppresses phase4 depolarization:e)shortens action potential:classIA,IB,IC,II,III,IVa)slows phase0 depolarization:IA,ICb)shortens phase3 repolarization:IBc)prolongs phase 3 repolarization:IIId)suppresses phase4 depolarization:II

Page 55: 2003 step 1

8/11/2019 2003 step 1

http://slidepdf.com/reader/full/2003-step-1 55/59

© usmile.us 

e)shortens action potential:IV

 A neuroscientist is investigating the development of the nervous system. In his experiments,he microinjects a dye into the embryo of an animal subject in vivo. After birth, he performshistological studies to determine the destination of the dye. In one animal subject, he locates thedye in the dorsal horn of the spinal cord. Which of the following locations in the embryo was themost likely site of the injection?

 A. Alar plateB. Basal plateC. Neural crestD. Rostral end of neural tube

E. Sulcus limitansa:alar plate

it contains tissue toxic bromide? A)halothaneB)methoxyfluraneC)ethyl etherD)benzodiazepines

E)NOF)isofluraneG)thiopentalhalothane

used solely in obstetric practice? A)halothaneB)methoxyfluraneC)ethyl etherD)benzodiazepinesE)NOF)isofluraneG)thiopental

methoxyflurane, not relax the uterus

which one of the following is most likely to require adminsteration of a muscle relaxant? A)halothaneB)methoxyfluraneC)ethyl etherD)benzodiazepinesE)NOit's NO

diastolic pressure is increased after the adminsteration of which one of the following drugs?a)norepinephrineb)epinephrine

c)isoproterenold)albuterole)terbutalineactually it affects both alpha&beta receptors,but alpha effect dominant

The ratio of urinary concentration to plasma concentration of inulin {(U/P) inulin} decreases.Which of the following is true if the glomerular filtration rate remains constant?

 A. Aldosterone levels have increasedB. Inulin clearance has decreasedC. Positive free water clearance has decreased

Page 56: 2003 step 1

8/11/2019 2003 step 1

http://slidepdf.com/reader/full/2003-step-1 56/59

Page 57: 2003 step 1

8/11/2019 2003 step 1

http://slidepdf.com/reader/full/2003-step-1 57/59

© usmile.us 

males:pseudohermaphrodism:ambigous genitaliafemales:primary amenorrhea

neurotransmitter deficient in Huntigton’s disease?GABA,Ach

rheumatoid factors originated from which class of Igs?IgM

in an identification of organisms,a medical student used Gram stain to differentiate betweengram+&gram- orgabisms,but he forgot to add iodine solution after crystal violet dye,whathappened after finishing the procedure?

steps of gr stain procedure:1)crystal violet dye stains all cells blue2)iodine solution is added to form a crystal violet-iodine complex,all cells continue to appear blue3)acetone or ethanol is added:gr- appear colorless,gr+remain blue4)red dye safranin stains gr-red,gr+remain blueif step2 omitted like in this case,bcoz acetone removes the violet-iodine complex not crystal violetalone:gr- STAIN BLUE,gr+ also BLUE,so u can't diff between them

Cloning of a new eukaryotic gene was followed by insertion and ligation of the gene in anexpression vector. The protein translated from this gene was then studied on a western blot, andprobed with 32P-DNA, yielding a positive result. These findings eliminate which of the followingsubstances as a candidate for the likely gene product?

 A. CREBB. Protein kinase AC. RNA polymeraseD. SP-1E. Steroid receptorB. The DNA probe can only detect a protein that is able to bind to DNA. Only protein kinase is nota DNA-binding protein. Others are transcription factors and RNA polymerase, which all bind toDNA.

the enzyme inhibited by mycophenolate mofetil and the clinical use of this inhibition?it inhibits inosine monophosphate dehydrogenase:an enzyme that converts IMPtoGMP,guanosine monophosphate required for nucleic acid synthesis in B,Tcells,so these cellsinhibited,it’s an immunosuppressive drug

the mechanism of action of corticostroids that’s responsible for their immunosuppressiveaction?Corticostroids dissociate nuclear factor kB(NF-kB)from its inhibitor(I-kB),inhibiting transcription ofcytokine genes.they also suppress IL-2 synthesis indirectly by blocking macrophage release of IL-1.

vasculitis with painful skin nodules?

Takayaso

the amount of ATP consumed&produced in HMP shunt?no ATP consumed or produced during HMP shunt!

what happens during embryonic development that leads to tetralogy of Fallot?anterosuperior displacement of infundibular septum;V septum is shifted from midline causing all features

2most common neoplasms in tuberous sclerosis?

Page 58: 2003 step 1

8/11/2019 2003 step 1

http://slidepdf.com/reader/full/2003-step-1 58/59

© usmile.us 

rhabdomyoma of the heart and angiomylypomas of the kidneysalso astrocytoma

alpha-1 antitrypsin a marker for which tumors?liver,yolk sac tumor (endodermal sinus tumor,hepatocellular carcinoma)

which type of gallstone seen in alcoholic cirrhosis?pigment stones

mechanism of action of cisplatin,bleomycin,adriamycin?No answer

rate-limiting enzyme in beta-oxidation of fatty acids&final product of FA beta-oxidation?CAT-I

 Acetyl coA –final productpalmitate final product of FA synthesis!

osteoprosis most common in which group?asians?whites?blacks?

whites>blacks>Asians

diuretic used to treat nephrogenic DI?both hydrochlorthizide&amiloride(acoording to FA)

Nagler reaction+ in culture environment indicative of which organism?it indicates presence of alpha toxin lecithinase: c.perfringens

thyroid dysfunction seen in treatment with:amiodarone?interferon?both hypothyroidism and hyperthyroidism can be a complication of treatment with amiodarone orIFN therapy

anti-endomesial Abs seen in which disease?in celiac

change in T3 resin uptake,TRH,TSH during pregnancy?since the TBGincreases the total T4 AND T3 UPTAKE WILL BE INCREASED all others arenormal

changes in arterial PO2,PCO2,PH and venous PCO2,V/Q ratio during exercisenormal arterial Po2, Pco2, PH, increased venous Pco2, V/Q close to 1Ph-no change initially, lactic acidosis during vig exercise

popcorn cells r charachteristic of which subtype of Hodgkin disease?

Lymphocyte predominant type of Hodgkins disease.

what change is seen in the level of LAP in myeloid metaplasia?increased LAP scoreCML decrease LAP score

these drugs associated with eosinophilia?drugs which coz interstitial nephritis?methicillindrugs that coz type 1 hypersensitivity reactions penicillin.

Page 59: 2003 step 1

8/11/2019 2003 step 1

http://slidepdf.com/reader/full/2003-step-1 59/59

© usmile.us 

Metabisulfite a diagnostic test for which hematologic disease?sickle cell

the most common location of medulloblastoma in adults&children?in kids-cerebellar vermis,ependymoma in the 4thventricle.in adults ?cerebellar hemispheres

cerebral tumor may contain progesterone receptors that responsible for its rapid growthduring pregnancy?meningioma

organism with tumbling motility?Tumbling motility- listeria

Swarming motility- proteus

the origin&role of thrombomedulin in coagulation system?Thrombomodulin (TM) is an integral membrane glycoprotein expressed on the surface ofendothelial cells. TM is an integral protein that contains one chondroitin/ dermatan sulphatechain.

It binds to thrombin and act as a cofactor in the activation of protein C catalyzed by thrombin. TMaccelerates the rate of protein C activation by 1000-fold relative to the rate with thrombin alone. Inaddition to facilitating protein C activation, the binding of thrombin to TM drastically alters theprocoagulant activity of thrombin. When bound to TM, thrombin no longer clots fibrinogen,activates factor V, inactivates protein S or triggers platelet aggregation.